[Physics] How are the Lorentz force, Maxwell’s third law and Faraday’s law of induction clasically related

electromagnetisminductionmagnetic fieldsmagnetic-monopolesmaxwell-equations

Faraday's law of induction can be used in any situation where the magnetic flux is changing through a closed conducting loop. While giving the correct answer, it seems to me that for the following scenarios different things are happening.

1) Let's consider a closed loop that is standing still in my reference frame. When changing the magnetic field through this loop from Maxwell 3, $\vec{\nabla} \times \vec{E} = -\frac{\partial \vec B}{\partial t}$, it is easy to find that Faraday's induction law is correct here using Stokes theorem. Physically, electric field lines are created inside the loop and push the electrons to move, this creates the current. The emf is then just the electric potential.

2) Let's place ourselves at rest relative to the magnet and change the area of the loop. $\frac{\partial \vec B}{\partial t}=0$ at any point in space so no $\vec{E}$ is created due to the field. This means that Maxwell 3 can't be really used in this frame. In this case the magnetic Lorentz force $F=q\vec{v} \times \vec{B}$ can be used to explain the movement of the electrons. For simple cases like these it seems to me that I have been succesful in deriving Faraday's induction law.

3) Finally I'd like to add that there is a thing called the disc of Faraday (http://en.wikipedia.org/wiki/Homopolar_generator) where no flux is changing but an emf is induced by the Lorentz force.

To conclude: I have a very basic understanding of special relativity and feel what is coming. I have to ask you to not go too deep into that for the following reason: I am studying for my classical electromagnetism final and need to have an efficient knowledge of the connections between laws from a classical point of view. Here come my two questions:

  • Is everything I said about the described scenarios correct?
  • What is the most efficient way to place this in my head in a classical way? Should I just consider Maxwell 3 and the Lorentz force being different phenomena that in problems where the flux is changing happen to lead to the same result, namely Faraday's induction?

Best Answer

I assume what is meant by Faraday's law of induction is what Griffiths refers to as the "universal flux rule", the statement of which can be found in this question. This covers both cases 1) and 2), even though in 1) it is justified by the third Maxwell equation1 and in 2) by the Lorentz force law.

The universal flux rule is a consequence of the third Maxwell equation, the Lorentz force law, and Gauss's law for magnetism (the second Maxwell equation). To the extent that those three laws are fundamental, the universal flux rule is not.

I won't comment on whether the universal flux rule is intuitively true. But the real relationship is given by the derivation of the universal flux rule from the Maxwell equations and the Lorentz force law. You can derive it yourself, but it requires you to either:

  1. know the form of the Leibniz integral rule for integration over an oriented surface in three dimensions
  2. be able to derive #1 from the more general statement using differential geometry
  3. be able to come up with an intuitive sort of argument involving infinitesimal deformations of the loop, like what is shown here.

If you look at the formula for (1), and set $\mathbf{F} = \mathbf{B}$, you see that \begin{align*} \frac{\mathrm{d}}{\mathrm{d}t} \iint_{\Sigma} \mathbf{B} \cdot \mathrm{d}\mathbf{a} &= \iint_{\Sigma} \dot{\mathbf{B}} \cdot \mathrm{d}\mathbf{a} + \iint_{\Sigma} \mathbf{v}(\nabla \cdot \mathbf{B}) \cdot \mathrm{d}\mathbf{a} - \int_{\partial \Sigma} \mathbf{v} \times \mathbf{B} \cdot \mathrm{d}\boldsymbol\ell \\ &= - \iint_{\Sigma} \nabla \times \mathbf{E} \cdot \mathrm{d}\mathbf{a} - \int_{\partial\Sigma} \mathbf{v} \times \mathbf{B} \cdot \mathrm{d}\boldsymbol\ell \\ &= -\int_{\partial \Sigma} \mathbf{E} + \mathbf{v} \times \mathbf{B} \cdot \mathrm{d}\boldsymbol\ell \end{align*}

where we have used the third Maxwell equation, Gauss's law for magnetism, and the Kelvin--Stokes theorem. The final expression on the right hand side is of course the negative emf in the loop, and we recover the universal flux rule.

Observe that the first term, $\iint_\Sigma \dot{\mathbf{B}} \cdot \mathrm{d}\mathbf{a}$, becomes the electric part of the emf, so if the loop is stationary and the magnetic field changes, then the resulting emf is entirely due to the induced electric field. In contrast, the third term, $-\int_{\partial\Sigma} \mathbf{v} \times \mathbf{B} \cdot \mathrm{d}\boldsymbol\ell$, becomes the magnetic part of the emf, so if the magnetic field is constant and the loop moves, then the resulting emf is entirely due to the Lorentz force. In general, when the magnetic field may change and the loop may also move simultaneously, the total emf is the sum of these two contributions.

If you are an undergrad taking a first course in electromagnetism, you should know the statement of the universal flux rule, and you should be able to justify it by working out specific cases using the third Maxwell equation, the Lorentz force law, or some combination thereof, but I can't imagine you would be asked for the proof of the general case from scratch, as given above.

The universal flux rule only applies to the case of an idealized wire, modelled as a continuous one-dimensional closed curve in which current is constrained to flow, that possibly undergoes a continuous deformation. It cannot be used for cases like the Faraday disc. In such cases you will need to go back to the first principles, that is, the third Maxwell equation and the Lorentz force law. There is no shortcut or generalization of the flux rule that you can apply. You should be able to do this on an exam.

1 This equation is also often referred to as "Faraday's law" (which I try to avoid) or the "Maxwell--Faraday equation/law" (which I will also avoid here because of the potential to cause confusion).